Top Banner
NOTE: Many features on this website require Javascript and cookies. You can enable both via your browser's preference settings. Dashboard Questions Scores Community Help Shop © 2010 BMJ Publishing Group Ltd . All rights reserved . Terms and conditions . Privacy policy . A A A Work smart Work hard Mock exams Revision advice Work smart: MRCPCH Part 1 B Question: 4 of 10 / Overall score: 60% A 7 day old baby presents with cyanosis. This is pathophysiologically due to abnormal mixing in the following conditions: True / False B, C, D 'Abnormal mixing' refers to those conditions where systemic and pulmonary circulations mingle. In Fallot's, cyanosis is caused by decreased pulmonary blood flow. Aortic coarctation does not cause cyanosis. Question supplied by Colin Melville Consultant Paediatrician Rate this question n m l k j n m l k j i Tetralogy of Fallot. Correct n m l k j i n m l k j Transposition of the great arteries. Correct n m l k j i n m l k j Total anomalous pulmonary venous drainage. Correct n m l k j i n m l k j Univentricular heart. Correct n m l k j i n m l k j Aortic coarctation. Incorrect answer selected Next question Sta Co W Question statistics Average score 70.17% Times answered 173 Average user rating 3 My account Sign out
79
Welcome message from author
This document is posted to help you gain knowledge. Please leave a comment to let me know what you think about it! Share it to your friends and learn new things together.
Transcript
Page 1: cadiology 1.pdf

NOTE: Many features on this website require Javascript and cookies. You can enable both via your browser's preference settings.

Dashboard

Questions

Scores

Community

Help

Shop

© 2010 BMJ Publishing Group Ltd . All rights reserved . Terms and conditions . Privacy policy . A A A

Work smart Work hard Mock exams Revision advice

 

Work smart: MRCPCH Part 1 B

Question: 4 of 10 / Overall score: 60%

A 7 day old baby presents with cyanosis. This is pathophysiologically due to abnormal mixing in the following conditions:

True / False

B, C, D 'Abnormal mixing' refers to those conditions where

systemic and pulmonary circulations mingle. In Fallot's,

cyanosis is caused by decreased pulmonary blood flow.

Aortic coarctation does not cause cyanosis.

Question supplied by Colin Melville Consultant Paediatrician

Rate this question

nmlkj nmlkji Tetralogy of Fallot. Correct

nmlkji nmlkj Transposition of the great arteries. Correct

nmlkji nmlkj Total anomalous pulmonary venous drainage. Correct

nmlkji nmlkj Univentricular heart. Correct

nmlkji nmlkj Aortic coarctation. Incorrect answer selected

Next question

 

Stats

 

Comments

 

Web links

Question statistics

Average score

70.17%

Times answered

173

Average user rating

3

 

My account Sign out

Page 2: cadiology 1.pdf

NOTE: Many features on this website require Javascript and cookies. You can enable both via your browser's preference settings.

Dashboard

Questions

Scores

Community

Help

Shop

© 2010 BMJ Publishing Group Ltd . All rights reserved . Terms and conditions . Privacy policy . A A A

Work smart Work hard Mock exams Revision advice

 

Work smart: MRCPCH Part 1 B

Question: 3 of 10 / Overall score: 53%

The following may be found in atrioventricular septal defects:

True / False

An RSR pattern is seen in primum ASD. An apical pan-systolic

murmur is common, and is caused by mitral regurgitation.

The second heart sound has wide fixed splitting. A

pulmonary flow murmur is usual.

eMedicine article

Question supplied by Colin Melville Consultant

Paediatrician

Rate this question

nmlkji nmlkj RSR1 pattern in V1. Correct

nmlkj nmlkji Mild cyanosis in the early stages. Correct

nmlkji nmlkj A pan-systolic murmur at the apex. Correct

nmlkj nmlkji Wide but variable splitting of the second heart sound. Correct

nmlkj nmlkji An ejection murmur at the upper left sternal border. Incorrect answer selected

Next question

 

Stats

 

Comments

 

Web links

Question statistics

Average score

57.61%

Times answered

176

Average user rating

4

 

My account Sign out

Page 3: cadiology 1.pdf

NOTE: Many features on this website require Javascript and cookies. You can enable both via your browser's preference settings.

Dashboard

Questions

Scores

Community

Help

Shop

© 2010 BMJ Publishing Group Ltd . All rights reserved . Terms and conditions . Privacy policy . A A A

Work smart Work hard Mock exams Revision advice

 

Work smart: MRCPCH Part 1 B

Question: 2 of 10 / Overall score: 40%

Regarding cardiogenic shock:

True / False

B, C The vagus slows the heart, and angiotensin II is

produced in the lungs. ANP production is increased in

response to atrial stretching, and generally the effects are

opposite to those of the renin-angiotensin-aldosterone

system.

Question supplied by Colin Melville Consultant Paediatrician

Rate this question

nmlkj nmlkji Increased firing of neurones in the carotid body stimulates the vagus nerve.

Correct

nmlkji nmlkj Renal cortical necrosis is a recognised complication. Correct

nmlkji nmlkj Constriction of the renal afferent arterioles stimulates renin production. Correct

nmlkji nmlkj Angiotensin II production is increased in the liver. Incorrect answer selected

nmlkji nmlkj ANP production is increased. Incorrect answer selected

Next question

 

Stats

 

Comments

 

Web links

Question statistics

Average score

63.20%

Times answered

175

Average user rating

4

 

My account Sign out

Page 4: cadiology 1.pdf

NOTE: Many features on this website require Javascript and cookies. You can enable both via your browser's preference settings.

Dashboard

Questions

Scores

Community

Help

Shop

© 2010 BMJ Publishing Group Ltd . All rights reserved . Terms and conditions . Privacy policy . A A A

Work smart Work hard Mock exams Revision advice

 

Work smart: MRCPCH Part 1 B

Question: 2 of 10 / Overall score: 40%

The following are recognised treatments for Fallot's spell:

True / False

The Fallot's spell is caused by a sudden increase in right

ventricular outflow tract obstruction caused by spasm of the

infundibulum. There is an acute decrease of blood circulating

around the lungs and the child becomes cyanosed, hypoxic,

and may loose consciousness. Treatment is 100% oxygen via

face mask, Morphine intramuscularly or intravenously, and/or

Propranolol. The knee-chest position is designed to increase

systemic vascular resistance and decrease shunting in the

heart, and Noradrenaline performs the same function. Although

Prostaglandin infusion may be useful in maintaining duct

patency during transfer to an acute cardiac centre, it is

unlikely to be of benefit in the acute situation.

Copyright 2002 Colin Melville

Rate this question

nmlkj nmlkji Noradrenaline infusion Incorrect answer selected

nmlkji nmlkj knee-chest position. Correct

nmlkji nmlkj Morphine 0.1mg/kg. Correct

nmlkj nmlkji Prostaglandin E2 50mg/kg/min. Correct

nmlkji nmlkj Propranolol 0.1mg/kg IV. Correct

Next question

 

Stats

 

Comments

 

Web links

Question statistics

Average score

65.56%

Times answered

180

Average user rating

4

 

My account Sign out

Page 5: cadiology 1.pdf

NOTE: Many features on this website require Javascript and cookies. You can enable both via your browser's preference settings.

Dashboard

Questions

Scores

Community

Help

Shop

© 2010 BMJ Publishing Group Ltd . All rights reserved . Terms and conditions . Privacy policy . A A A

Work smart Work hard Mock exams Revision advice

 

Work smart: MRCPCH Part 1 B

Question: 1 of 10 / Overall score: 20%

In the fetal circulation

True / False

In the fetal circulation, the vast majority of blood bypasses

the lungs through the ductus arteriosus, but the right and left

ventricles pump approximately equal volumes. The fetus is

relatively hypoxaemic, with fetal haemoglobin being used to

increase oxygen carriage to the tissues. Only 10% of cardiac

output traverses the fetal lung, and the ductus venosus

remains patent until the postnatal period. Approximately 50%

of umbilical venous catheters can be passed through it into the

right atrium postnatally. The pulmonary-vascular resistance

remains suprasystemic until the time of the first breath, when

the combination of lung aeration reducing pulmonary blood

pressure, and umbilical ligation increasing systemic blood

pressure reverses the situation.

Copyright 2002 Colin Melville

Rate this question

nmlkj nmlkji Approximately equal volumes of blood are pumped by the left and right ventricles.

Incorrect answer selected

nmlkj nmlkji The saturation of blood returning from the placenta is about 80%. Incorrect answer selected

nmlkji nmlkj Approximately 25% of cardiac output traverses the fetal lung. Incorrect answer selected

nmlkji nmlkj The ductus venosus is probe-patent in 75% of cases. Incorrect answer selected

nmlkji nmlkj Pulmonary vascular resistance is suprasystemic. Correct

Next question

 

Stats

 

Comments

 

Web links

Question statistics

Average score

54.65%

Times answered

172

Average user rating

4

 

My account Sign out

Page 6: cadiology 1.pdf

NOTE: Many features on this website require Javascript and cookies. You can enable both via your browser's preference settings.

Dashboard

Questions

Scores

Community

Help

Shop

© 2010 BMJ Publishing Group Ltd . All rights reserved . Terms and conditions . Privacy policy . A A A

Work smart Work hard Mock exams Revision advice

 

Work smart: MRCPCH Part 1 B

Question: 19 of 40 / Overall score: 64%

The following lesions can be diagnosed antenatally on a routine 4 chamber view at 18 weeks gestation:

True / False

The routine 4 chamber view is likely to pick up only severe

lesions causing small right or left ventricles, such as

hypoplastic left heart syndrome or pulmonary atresia.

Copyright 2002 Colin Melville

Rate this question

nmlkji nmlkj Tetralogy of Fallot. Incorrect answer selected

nmlkji nmlkj Hypoplastic left heart syndrome. Correct

nmlkj nmlkji VSD. Correct

nmlkj nmlkji Pulmonary stenosis. Correct

nmlkji nmlkj Transposition of the great arteries. Incorrect answer selected

Next question

 

Stats

 

Comments

 

Web links

Question statistics

Average score

46.42%

Times answered

165

Average user rating

3

 

My account Sign out

Page 7: cadiology 1.pdf

NOTE: Many features on this website require Javascript and cookies. You can enable both via your browser's preference settings.

Dashboard

Questions

Scores

Community

Help

Shop

© 2010 BMJ Publishing Group Ltd . All rights reserved . Terms and conditions . Privacy policy . A A A

Work smart Work hard Mock exams Revision advice

 

Work smart: MRCPCH Part 1 B

Question: 18 of 40 / Overall score: 64%

An infant is noted at postnatal examination to have a slow pulse rate. ECG confirms congenital complete heart block. The following statements are true:

True / False

The commonest cause is maternal SLE, which may be

asymptomatic, and is associated with anti-Ro antibodies.

These were found in 34/41 mothers who delivered children with

CHB. Echocardiography is usually normal (>60% of cases), and

heart failure is uncommon. Maternal hyperthyroidism can cause

abortion, preterm labour, and fetal tachycardia. Prolonged QT

may occur with hypokalaemia, hypocalcaemia, antihistamines

(e.g. Terfenadine). Congenital prolongation presents with

syncope in late childhood.

Copyright 2002 Colin Melville

Rate this question

nmlkji nmlkj Echocardiography is usually normal. Correct

nmlkji nmlkj It is usually associated with a presence of anti-Ro antibodies in the mother. Correct

nmlkj nmlkji Heart failure is uncommon. Incorrect answer selected

nmlkji nmlkj Mother's thyroid function should be checked. Incorrect answer selected

nmlkji nmlkj The underlying defect may be due to prolonged QT Syndrome. Incorrect answer selected

Next question

 

Stats

 

Comments

 

Web links

Question statistics

Average score

53.29%

Times answered

167

Average user rating

4

 

My account Sign out

Page 8: cadiology 1.pdf

NOTE: Many features on this website require Javascript and cookies. You can enable both via your browser's preference settings.

Dashboard

Questions

Scores

Community

Help

Shop

© 2010 BMJ Publishing Group Ltd . All rights reserved . Terms and conditions . Privacy policy . A A A

Work smart Work hard Mock exams Revision advice

 

Work smart: MRCPCH Part 1 B

Question: 17 of 40 / Overall score: 66%

The following are features of aortic coarctation in infancy:

True / False

B, C, D Rib notching is a late sign in adolescents, and is rarely

seen these days. An inverted T wave in V6 suggests left

ventricular strain. Plateau pulses are a feature of aortic

stenosis. There may be a systolic murmur along the left sternal

border with a loud 2nd heart sound.

Copyright 2002 Colin Melville

Rate this question

nmlkj nmlkji Rib-notching. Correct

nmlkji nmlkj Inverted T wave in V6. Correct

nmlkji nmlkj Ejection systolic murmur between the shoulder blades. Incorrect answer selected

nmlkji nmlkj Radio-femoral delay. Correct

nmlkj nmlkji Plateau pulses. Correct

Next question

 

Stats

 

Comments

 

Web links

Question statistics

Average score

56.02%

Times answered

166

Average user rating

3

 

My account Sign out

Page 9: cadiology 1.pdf

NOTE: Many features on this website require Javascript and cookies. You can enable both via your browser's preference settings.

Dashboard

Questions

Scores

Community

Help

Shop

© 2010 BMJ Publishing Group Ltd . All rights reserved . Terms and conditions . Privacy policy . A A A

Work smart Work hard Mock exams Revision advice

 

Work smart: MRCPCH Part 1 B

Question: 16 of 40 / Overall score: 65%

The following are recognised features of cardiac failure in the infant:

True / False

Cardiac failure in the infant usually presents with feeding

difficulties and respiratory distress. On examination, there may

be sweating, tachypnoea, nasal flaring, recession, grunt,

hepatomegaly, and splenomegaly. Pallor may reflect poor

peripheral perfusion, and there may be a murmur from the

underlying heart lesion. The squat neck of the infant makes

the jugular venous pulse impossible to assess reliably, and

jaundice does not occur.

Copyright 2002 Colin Melville

Rate this question

nmlkj nmlkji Jaundice. Correct

nmlkj nmlkji Splenomegaly. Incorrect answer selected

nmlkj nmlkji Clinically detectable elevation jugular venous pulse. Correct

nmlkji nmlkj Pallor. Correct

nmlkji nmlkj Grunting. Correct

Next question

 

Stats

 

Comments

 

Web links

Question statistics

Average score

73.26%

Times answered

178

Average user rating

3

 

My account Sign out

Page 10: cadiology 1.pdf

NOTE: Many features on this website require Javascript and cookies. You can enable both via your browser's preference settings.

Dashboard

Questions

Scores

Community

Help

Shop

© 2010 BMJ Publishing Group Ltd . All rights reserved . Terms and conditions . Privacy policy . A A A

Work smart Work hard Mock exams Revision advice

 

Work smart: MRCPCH Part 1 B

Question: 15 of 40 / Overall score: 64%

Interventional catheterisation can be used to repair the following defects:

True / False

B, D Ostium primum ASD is at the minor end of the spectrum of

AV canal defects. Perimembranous VSDs require surgical

closure, but muscular ones often close themselves. Dysplastic

pulmonary valves can be palliated by balloon dilatation, but

usually require surgical repair later. Secundum ASDs and PDA,

can by closed using umbrella devices.

Copyright 2002 Colin Melville

Rate this question

nmlkji nmlkj Ostium primum ASD. Incorrect answer selected

nmlkji nmlkj Ostium secundum ASD. Correct

nmlkj nmlkji Muscular VSD. Correct

nmlkji nmlkj PDA. Correct

nmlkji nmlkj Dysplastic pulmonary valve. Incorrect answer selected

Next question

 

Stats

 

Comments

 

Web links

Question statistics

Average score

60.70%

Times answered

171

Average user rating

4

 

My account Sign out

Page 11: cadiology 1.pdf

NOTE: Many features on this website require Javascript and cookies. You can enable both via your browser's preference settings.

Dashboard

Questions

Scores

Community

Help

Shop

© 2010 BMJ Publishing Group Ltd . All rights reserved . Terms and conditions . Privacy policy . A A A

Work smart Work hard Mock exams Revision advice

 

Work smart: MRCPCH Part 1 B

Question: 14 of 40 / Overall score: 64%

Myocarditis is a recognised association of:

True / False

A, B, C, D Hurler's is associated with a cardiomyopathy.

Copyright 2002 Colin Melville

Rate this question

nmlkji nmlkj Salmonella infection. Correct

nmlkj nmlkji Mycoplasma infection. Incorrect answer selected

nmlkj nmlkji Lyme Disease. Incorrect answer selected

nmlkji nmlkj Measles. Correct

nmlkji nmlkj Hurler's Syndrome. Incorrect answer selected

Next question

 

Stats

 

Comments

 

Web links

Question statistics

Average score

65.17%

Times answered

174

Average user rating

5

 

My account Sign out

Page 12: cadiology 1.pdf

NOTE: Many features on this website require Javascript and cookies. You can enable both via your browser's preference settings.

Dashboard

Questions

Scores

Community

Help

Shop

© 2010 BMJ Publishing Group Ltd . All rights reserved . Terms and conditions . Privacy policy . A A A

Work smart Work hard Mock exams Revision advice

 

Work smart: MRCPCH Part 1 B

Question: 13 of 40 / Overall score: 66%

Recognised features of a large uncomplicated patent ductus arteriosus include:

True / False

B, D A large uncomplicated patent ductus arteriosus results in

a bounding pulse and wide pulse pressure, because of

increased cardiac output on the left side of the heart draining

to a low resistance pulmonary circulation. Because of

increased left to right shunt, pulmonary plethora and

cardiomegaly are characteristic. Squatting will increase

systemic arterial resistance, but this is unlikely to have any

effect on the murmur. Because the left ventricle pumping at

excessive volume, left ventricular hypertrophy and strain may

be seen on the ECG. The second pulmonary heart sound is loud

because of pulmonary hypertension, but may not be audible

beneath a loud murmur.

Copyright 2002 Colin Melville

Rate this question

nmlkj nmlkji Narrow pulse pressure. Correct

nmlkj nmlkji Pulmonary plethora. Incorrect answer selected

nmlkj nmlkji Decreased noise of murmur on squatting. Correct

nmlkj nmlkji Left ventricular hypertrophy on ECG. Incorrect answer selected

nmlkji nmlkj Soft pulmonary second heart sound. Incorrect answer selected

Next question

 

Stats

 

Comments

 

Web links

Question statistics

Average score

65.43%

Times answered

173

Average user rating

4

 

My account Sign out

Page 13: cadiology 1.pdf

NOTE: Many features on this website require Javascript and cookies. You can enable both via your browser's preference settings.

Dashboard

Questions

Scores

Community

Help

Shop

© 2010 BMJ Publishing Group Ltd . All rights reserved . Terms and conditions . Privacy policy . A A A

Work smart Work hard Mock exams Revision advice

 

Work smart: MRCPCH Part 1 B

Question: 9 of 10 / Overall score: 87%

Do the following suggest pulmonary rather than aortic

valve stenosis?

True / False

An ejection murmur radiating to the back (along the

direction of the pulmonary arteries) is found in pulmonary

stenosis. The ejection click is found at the apex in aortic

stenosis.

A deep S wave in V2 and large R wave in V6 suggests left

ventricular hypertrophy (LVH), while an upright T wave in

V1 suggests right ventricular hypertrophy (RVH).

A prominent left upper mediastinum on chest x ray may be

caused by post-stenotic aortic dilatation in aortic stenosis.

Copyright 2002 Colin Melville

Rate this question

nmlkji nmlkj A deep S wave in V2 Incorrect answer selected

nmlkji nmlkj An ejection click best heard in the second and third left intercostal spaces Correct

nmlkj nmlkji An ejection murmur radiating to the neck Correct

nmlkj nmlkji An upright T wave in V1 Incorrect answer selected

nmlkji nmlkj Prominence in the left upper mediastinum on chest x ray Incorrect answer selected

Next question

 

Stats

 

Comments

 

Web links

Question statistics

Average score

66.43%

Times answered

199

Average user rating

3

 

My account Sign out

Page 14: cadiology 1.pdf

NOTE: Many features on this website require Javascript and cookies. You can enable both via your browser's preference settings.

Dashboard

Questions

Scores

Community

Help

Shop

© 2010 BMJ Publishing Group Ltd . All rights reserved . Terms and conditions . Privacy policy . A A A

Work smart Work hard Mock exams Revision advice

 

Work smart: MRCPCH Part 1 B

Question: 12 of 40 / Overall score: 68%

The following are indications for immediate transfer to a tertiary cardiology centre in a neonate:

True / False

E Urgent transfer to a tertiary cardiac centre is required for

life-threatening conditions, particularly those dependent on the

ductus arteriosus for survival. These include: 1. Hypoplastic

left heart syndrome. 2. Critical aortic valve stenosis. 3.

Coarctation of the aorta. 4. Interruption of the aortic arch.

The other conditions mentioned require more routine cardiac

evaluation.

Copyright 2002 Colin Melville

Rate this question

nmlkji nmlkj Suspected large VSD. Incorrect answer selected

nmlkj nmlkji Suspected ASD. Correct

nmlkj nmlkji A diagnosis of Down's Syndrome. Correct

nmlkji nmlkj A diagnosis of Kawasaki Disease. Incorrect answer selected

nmlkji nmlkj Suspected aortic coarctation. Correct

Next question

 

Stats

 

Comments

 

Web links

Question statistics

Average score

77.14%

Times answered

182

Average user rating

4

 

My account Sign out

Page 15: cadiology 1.pdf

NOTE: Many features on this website require Javascript and cookies. You can enable both via your browser's preference settings.

Dashboard

Questions

Scores

Community

Help

Shop

© 2010 BMJ Publishing Group Ltd . All rights reserved . Terms and conditions . Privacy policy . A A A

Work smart Work hard Mock exams Revision advice

 

Work smart: MRCPCH Part 1 B

Question: 11 of 40 / Overall score: 69%

Broad complex tachycardias:

True / False

Wide complex tachycardias may be due to ventricular

fibrillation, ventricular tachycardia, or supraventricular

tachycardia with associated ventricular conduction defect.

It may therefore be associated with shock and

pulselessness. Unstable patients should therefore be treated

as if they have ventricular tachycardia. Adenosine will

distinguish the rare SVT with conduction defect, and

Lidocaine can be tried in VT if no shock is present.

Otherwise (and usually) synchronous DC shock 0.5J/kg is

used.

Copyright 2002 Colin Melville

Rate this question

nmlkji nmlkj May be associated with shock. Correct

nmlkji nmlkj May be associated with pulselessness Correct

nmlkji nmlkj In the shocked patient are usually treated with synchronous DC shock. Correct

nmlkj nmlkji Are uncommon in childhood. Incorrect answer selected

nmlkj nmlkji May be due to supraventricular tachycardia. Incorrect answer selected

Next question

 

Stats

 

Comments

 

Web links

Question statistics

Average score

73.30%

Times answered

182

Average user rating

3

 

My account Sign out

Page 16: cadiology 1.pdf

NOTE: Many features on this website require Javascript and cookies. You can enable both via your browser's preference settings.

Dashboard

Questions

Scores

Community

Help

Shop

© 2010 BMJ Publishing Group Ltd . All rights reserved . Terms and conditions . Privacy policy . A A A

Work smart Work hard Mock exams Revision advice

 

Work smart: MRCPCH Part 1 B

Question: 10 of 40 / Overall score: 70%

Regarding infective endocarditis:

True / False

B, C, D Infective endocarditis is usually caused by Strep.

Viridans (Group D), but Staphylococcus aureus is becoming

increasingly common. The portal of entry is usually the teeth.

The organisms settle usually on congenital or rheumatic heart

lesions, particularly in areas of high velocity blood flow. Early

symptoms and signs are usually mild, and include prolonged

fever and weight loss, which may last for several months

before diagnosis. A rare presentation is with high fever and

prostration, but the usual course is somewhere between these

two. Fever, fatigue, myalgia, changing heart murmurs, heart

failure, splenomegaly and petechia are common. Serious

complications include cerebral abscess and mycotic aneurysms.

Osler's nodes, Janeway lesions and splinter haemorrhages may

occur due to vasculitis.

Copyright 2002 Colin Melville

Rate this question

nmlkj nmlkji The commonest causative organism is Group A Streptococcus. Correct

nmlkji nmlkj Lesions are most frequent in areas of high velocity blood flow. Correct

nmlkji nmlkj Amoxicillin is first choice for prophylaxis in procedures done under local anaesthetic.

Correct

nmlkj nmlkji Presentation in infancy is extremely rare. Incorrect answer selected

nmlkj nmlkji Splenomegaly is a subtle early sign. Correct

Next question

 

Stats

 

Comments

 

Web links

Question statistics

Average score

59.12%

Times answered

182

Average user rating

4

 

My account Sign out

Page 17: cadiology 1.pdf

NOTE: Many features on this website require Javascript and cookies. You can enable both via your browser's preference settings.

Dashboard

Questions

Scores

Community

Help

Shop

© 2010 BMJ Publishing Group Ltd . All rights reserved . Terms and conditions . Privacy policy . A A A

Work smart Work hard Mock exams Revision advice

 

Work smart: MRCPCH Part 1 B

Question: 9 of 40 / Overall score: 69%

The following findings suggest that a murmur is innocent:

True / False

About 30% of children can have innocent murmurs. These are

usually ejection generated by the outflow tracts of the left of

right side of the heart, or venous hums due to turbulent flow in

the head and neck veins. The hallmarks are their localised

nature; the fact they are confined to systole; the fact they

are soft; and that they are associated with no symptoms or

other signs.

Copyright 2002 Colin Melville

Rate this question

nmlkj nmlkji It is grade 3/6 Correct

nmlkji nmlkj It has a vibratory quality. Correct

nmlkj nmlkji It is loudest at the apex. Correct

nmlkj nmlkji It is only heard when the patient is febrile. Incorrect answer selected

nmlkji nmlkj It changes in intensity with posture. Correct

Next question

 

Stats

 

Comments

 

Web links

Question statistics

Average score

76.63%

Times answered

184

Average user rating

4

 

My account Sign out

Page 18: cadiology 1.pdf

NOTE: Many features on this website require Javascript and cookies. You can enable both via your browser's preference settings.

Dashboard

Questions

Scores

Community

Help

Shop

© 2010 BMJ Publishing Group Ltd . All rights reserved . Terms and conditions . Privacy policy . A A A

Work smart Work hard Mock exams Revision advice

 

Work smart: MRCPCH Part 1 B

Question: 8 of 40 / Overall score: 68%

The following are recognised causes of cyanosis in the newborn:

True / False

Cyanosis can be caused by non-cardiac or cardiac lesions. The

former include: persistent fetal circulation, hyaline membrane

disease, congenital pneumonia and transient tachypnoea of

the newborn. The latter include lesions due to abnormal mixing

(TGA, univentricular heart) or to decrease pulmonary blood

flow (pulmonary atresia, Fallot's Tetralogy). Hypoplastic left

heart syndrome can lead to severe congestive cardiac failure,

shock and secondary cyanosis.

Copyright 2002 Colin Melville

Rate this question

nmlkji nmlkj Persistent fetal circulation. Correct

nmlkj nmlkji Severe anaemia. Correct

nmlkj nmlkji Congenital pneumonia. Incorrect answer selected

nmlkj nmlkji Transient tachypnoea of the newborn. Incorrect answer selected

nmlkji nmlkj Hypoplastic left heart syndrome. Correct

Next question

 

Stats

 

Comments

 

Web links

Question statistics

Average score

66.02%

Times answered

186

Average user rating

4

 

My account Sign out

Page 19: cadiology 1.pdf

NOTE: Many features on this website require Javascript and cookies. You can enable both via your browser's preference settings.

Dashboard

Questions

Scores

Community

Help

Shop

© 2010 BMJ Publishing Group Ltd . All rights reserved . Terms and conditions . Privacy policy . A A A

Work smart Work hard Mock exams Revision advice

 

Work smart: MRCPCH Part 1 B

Question: 7 of 40 / Overall score: 69%

Nitric oxide (endothelial relaxation factor):

True / False

Prostaglandin I2 inhibits platelet aggregation via cAMP, while

nitric oxide works via cGMP. Glycerol trinitrate and

nitroprusside are converted to nitric oxide before having

their pharmacological effects. Nitric oxide is synthesised by

macrophages, lymphocytes, endothelial cells, and is a

neurotransmitter and vasodilator. When given by inhalation it

is a specific pulmonary vasodilator because of its short half

life.

Copyright 2002 Colin Melville

Rate this question

nmlkj nmlkji Unlike prostacyclin, promotes platelet aggregation. Correct

nmlkji nmlkj Can be enzymatically synthesised from glyceryl trinitrate. Correct

nmlkj nmlkji Has a half life of 1-2 minutes in vivo. Correct

nmlkji nmlkj Can be synthesised by macrophages. Correct

nmlkji nmlkj Is synthesised from L-arginine. Correct

Next question

 

Stats

 

Comments

 

Web links

Question statistics

Average score

65.48%

Times answered

186

Average user rating

4

 

My account Sign out

Page 20: cadiology 1.pdf

NOTE: Many features on this website require Javascript and cookies. You can enable both via your browser's preference settings.

Dashboard

Questions

Scores

Community

Help

Shop

© 2010 BMJ Publishing Group Ltd . All rights reserved . Terms and conditions . Privacy policy . A A A

Work smart Work hard Mock exams Revision advice

 

Work smart: MRCPCH Part 1 B

Question: 6 of 40 / Overall score: 63%

Paradoxical (reverse) splitting of the second heart sound is typical of:

True / False

C, D Paradoxical (reverse) splitting of the second heart sound

occurs when splitting sounds larger in expiration than

inspiration. Normally, inspiration increases right-sided filling and

delays pulmonary valve closure. In situations such as aortic

stenosis, or left bundle branch block, delayed closure of the

aortic valve results in reverse splitting.

Copyright 2002 Colin Melville

Rate this question

nmlkj nmlkji Ostium primum ASD. Correct

nmlkj nmlkji Pulmonary stenosis. Correct

nmlkji nmlkj Aortic stenosis. Correct

nmlkji nmlkj Left bundle branch block. Correct

nmlkj nmlkji Ostium secundum ASD. Correct

Next question

 

Stats

 

Comments

 

Web links

Question statistics

Average score

54.92%

Times answered

181

Average user rating

3

 

My account Sign out

Page 21: cadiology 1.pdf

NOTE: Many features on this website require Javascript and cookies. You can enable both via your browser's preference settings.

Dashboard

Questions

Scores

Community

Help

Shop

© 2010 BMJ Publishing Group Ltd . All rights reserved . Terms and conditions . Privacy policy . A A A

Work smart Work hard Mock exams Revision advice

 

Work smart: MRCPCH Part 1 B

Question: 5 of 40 / Overall score: 56%

Diastolic dysfunction of the left ventricle may be a dominant feature of:

True / False

A, B, C, D, E Diastolic dysfunction is a particular feature of

anthracycline chemotherapy, iron overload, and heart

rejection following transplantation. It may also be seen in

cardiac ischaemia.

Copyright 2002 Colin Melville

Rate this question

nmlkji nmlkj Hypertrophic obstructive cardiomyopathy. Correct

nmlkji nmlkj Endocardial fibroelastosis. Correct

nmlkj nmlkji Doxorubicin toxicity. Incorrect answer selected

nmlkji nmlkj Anomalous left coronary artery. Correct

nmlkj nmlkji Myocardial infarction after Kawasaki Disease. Incorrect answer selected

Next question

 

Stats

 

Comments

 

Web links

Question statistics

Average score

69.06%

Times answered

192

Average user rating

3

 

My account Sign out

Page 22: cadiology 1.pdf

NOTE: Many features on this website require Javascript and cookies. You can enable both via your browser's preference settings.

Dashboard

Questions

Scores

Community

Help

Shop

© 2010 BMJ Publishing Group Ltd . All rights reserved . Terms and conditions . Privacy policy . A A A

Work smart Work hard Mock exams Revision advice

 

Work smart: MRCPCH Part 1 B

Question: 4 of 40 / Overall score: 55%

Regarding supraventricular tachycardias:

True / False

All answers are false. SVT is usually due to re-entry within

the AV node due to an accessory pathway, and is

characterised clinically by abrupt onset and cessation. The

heart rate exceeds 180 beats per minute, and can

occasionally be as fast as 300 beats per minute. The heart

is usually anatomically normal, but there may be an

associated bypass tract in WPW or Lown-Ganong-Levine

Syndromes. It may also occur in relation to Ebstein's

anomaly or corrected transposition of the great arteries. In

older children it can be precipitated by sympathomimetics

such as cold cures. Vagal manoeuvres or facial immersion in

ice may abort the attack. Adenosine may be used in the

non-shocked patient, or DC cardio-aversion 0.5J/kg in the

child with congestive heart failure.

Copyright 2002 Colin Melville

Rate this question

nmlkj nmlkji Rates of 120-180 beats per minute are typical in adolescence. Correct

nmlkj nmlkji They are usually associated with structural cardiac defects. Correct

nmlkj nmlkji Lown-Ganong-Levine Syndrome is the most commonest cause. Correct

nmlkji nmlkj J waves may be seen. Incorrect answer selected

nmlkj nmlkji DC shock is most commonly required. Correct

Next question

 

Stats

 

Comments

 

Web links

Question statistics

Average score

79.69%

Times answered

192

Average user rating

4

 

My account Sign out

Page 23: cadiology 1.pdf

NOTE: Many features on this website require Javascript and cookies. You can enable both via your browser's preference settings.

Dashboard

Questions

Scores

Community

Help

Shop

© 2010 BMJ Publishing Group Ltd . All rights reserved . Terms and conditions . Privacy policy . A A A

Work smart Work hard Mock exams Revision advice

 

Work smart: MRCPCH Part 1 B

Question: 3 of 40 / Overall score: 47%

Regarding systemic hypertension in childhood:

True / False

D Sodium nitroprusside is useful only in the short term, as

cyanide levels accumulate with time. Hypertension is usually

diagnosed incidentally, and is defined as systolic blood

pressure >95th centile for age. Secondary causes are usually

due to renal abnormalities, with reflux associated scarring

being the commonest renal disease. This will cause

abnormalities on DMSA scan. Coarctation of the aorta is the

commonest non-renal cause, with

pheochromocytoma/neuroblastoma, congenital adrenal

hyperplasia, Cushing Syndrome and steroid therapy being rarer

causes.

Copyright 2002 Colin Melville

Rate this question

nmlkj nmlkji Sodium nitroprusside is useful for the long-term treatment of severe cases. Correct

nmlkj nmlkji Headache is the usual presenting feature. Correct

nmlkj nmlkji It is defined as systolic blood pressure above the 99th centile for age. Correct

nmlkji nmlkj Abnormalities are frequently seen on DMSA scan. Correct

nmlkji nmlkj Aortic coarctation is the commonest secondary cause. Incorrect answer selected

Next question

 

Stats

 

Comments

 

Web links

Question statistics

Average score

59.79%

Times answered

192

Average user rating

4

 

My account Sign out

Page 24: cadiology 1.pdf

NOTE: Many features on this website require Javascript and cookies. You can enable both via your browser's preference settings.

Dashboard

Questions

Scores

Community

Help

Shop

© 2010 BMJ Publishing Group Ltd . All rights reserved . Terms and conditions . Privacy policy . A A A

Work smart Work hard Mock exams Revision advice

 

Work smart: MRCPCH Part 1 B

Question: 9 of 10 / Overall score: 72%

A 3 month old girl presents with apnoea. She had been well

that morning, but had become unsettled, crying

inconsolably and gradually more mottled. Mother was

bringing her to A and E when she stopped breating. She

responded to physical stimulation. She was born at

40+3/40 weighing 3.6kg and there were no neonatal

problems.

On examination she has a temperature of 36.3&degC

(tympanic), RR 30/min and HR of 240/min. Her pulse in

thready. She has a 3 cm liver and gallop rhythm.

What is the most likely diagnosis?

(Please select 1 option)

The history suggests apnoea precipitated by

tachyarrhythmia. This is most likely to be a supraventricular

tachycardia. This can be confirmed by ECG monitoring, and

is usually successfully reverted by adenosine with digoxin

maintenance therapy. An echocardiogram will exclude the

rare possibility of an underlying structural defect.

Rate this question

nmlkj Acute life-threatening event

nmlkji Cardiac dysrhythmias Correct

nmlkj Seizures

nmlkj Sudden infant death syndrome

Next question

 

Stats

 

Comments

 

Web links

Question statistics

Option analysis

Times answered

207

Average user rating

3

 

1

2

3

4

24%

75%

0%

0%

My account Sign out

Page 25: cadiology 1.pdf

NOTE: Many features on this website require Javascript and cookies. You can enable both via your browser's preference settings.

Dashboard

Questions

Scores

Community

Help

Shop

© 2010 BMJ Publishing Group Ltd . All rights reserved . Terms and conditions . Privacy policy . A A A

Work smart Work hard Mock exams Revision advice

 

Work smart: MRCPCH Part 1 B

Question: 8 of 10 / Overall score: 68%

A 2-year-old child has an uncomplicated coarctation of the aorta. The constriction is located distal to the left subclavian artery. Which of the following would be decreased in this patient?

(Please select 1 option)

This patient with a fully compensated coarctation blood flow is

normal in both the lower and upper limbs despite increased

pressure in the upper limbs compared to lower body. Thus as

resistance=pressure/blood flow, resistance must be lower in

the lower limbs.

Rate this question

nmlkj Blood flow in the lower body

nmlkj Blood flow in the upper body

nmlkj Blood pressure in the upper limbs

nmlkj Vascular resistance in the lower limbs This is the correct answer

nmlkji Vascular resistance in the upper limbs Incorrect answer selected

Next question

 

Stats

 

Comments

 

Web links

Question statistics

Option analysis

Times answered

260

Average user rating

5

 

1

2

3

4

5

64%

3%

12%

15%

5%

My account Sign out

Page 26: cadiology 1.pdf

NOTE: Many features on this website require Javascript and cookies. You can enable both via your browser's preference settings.

Dashboard

Questions

Scores

Community

Help

Shop

Work smart Work hard Mock exams Revision advice

 

Work smart: MRCPCH Part 1 B

Question: 7 of 10 / Overall score: 78%

Theme: Genetics - cardiac abnormalities in genetic disorders

Match each of the following cardiovascular abnormalities to the single most likely associated genetic disorder.

Dilation of the aorta with aneurysms.

Incorrect - The correct answer is Marfan's syndrome

In Marfan's syndrome dilatation of the ascending aorta is often

seen with or without aneurysms. Less commonly the thoracic

abdominal aorta or pulmonary arteries are affected with

secondary aortic regurgitation and mitral valve prolapse.

Supra-valvular aortic stenosis.

Correct

In Williams syndrome supra-valvular aortic stenosis is the most

common cardiac lesion. Septal defects also occur as well as

peripheral branch pulmonary artery stenosis.

Pulmonary stenosis.

Correct

In Noonan's syndrome pulmonary valve stenosis due to a

dysplastic or thick valve is seen often associated with left

ventricular hypertrophy. Branch stenosis of the pulmonary

artery also is found in Noonan's syndrome.

A Angelman's syndrome

B Beckwith-Wiedemann syndrome

C Congenital Rubella syndrome

D Down syndrome

E Foetal alcohol syndrome

F Glycogen storage disease

G Marfan's syndrome

H Noonan's syndrome

I Turner's syndrome

J Williams syndrome

66Angelman's syndrome

66Williams syndrome

66Noonan's syndrome

 

Stats

 

Comments

 

Web links

Question statistics

Average score

79.44%

Times answered

201

Average user rating

4

 

My account Sign out

Page 27: cadiology 1.pdf

© 2010 BMJ Publishing Group Ltd . All rights reserved . Terms and

conditions . Privacy policy . A A A

Rate this question

Next question

Page 28: cadiology 1.pdf

NOTE: Many features on this website require Javascript and cookies. You can enable both via your browser's preference settings.

Dashboard

Questions

Scores

Community

Help

Shop

Work smart Work hard Mock exams Revision advice

 

Work smart: MRCPCH Part 1 B

Question: 6 of 10 / Overall score: 80%

Theme: Congenital cardiac defects

Match each of the following cardiovascular abnormalities to the single most likely associated disorder.

Endocardial cushion defect.

Incorrect - The correct answer is Down syndrome

In Down syndrome approximately 40% of children have a

congenital heart disease. The most common being endocardial

cushion defects although VSDs, ASDs and PDA also occur.

Coarctation of the aorta.

Correct

In Turner's syndrome cardiac defects are common. 30% include

bicuspid aortic valves with the second most common heart

defect being coarctation of the aorta. Aortic stenosis, mitral

valve prolapse and hypertension are also found.

Septal defects.

Incorrect - The correct answer is Foetal alcohol syndrome

In foetal alcohol syndrome individuals have poor growth,

developmental delay and usually characteristic facial features

including microcephaly and a short smooth philtrum. The most

common cardiac lesion in these children are septal defects

primarily ventricular septal defects.

A Angelman's syndrome

B Beckwith-Wiedemann syndrome

C Congenital Rubella syndrome

D Down syndrome

E Foetal alcohol syndrome

F Glycogen storage disease

G Marfan's syndrome

H Noonan's syndrome

I Turner's syndrome

J Williams syndrome

66Angelman's syndrome

66Turner's syndrome

66Down syndrome

 

Stats

 

Comments

 

Web links

Question statistics

Average score

64.16%

Times answered

203

Average user rating

4

 

My account Sign out

Page 29: cadiology 1.pdf

© 2010 BMJ Publishing Group Ltd . All rights reserved . Terms and

conditions . Privacy policy . A A A

Rate this question

Next question

Page 30: cadiology 1.pdf

NOTE: Many features on this website require Javascript and cookies. You can enable both via your browser's preference settings.

Dashboard

Questions

Scores

Community

Help

Shop

Work smart Work hard Mock exams Revision advice

 

Work smart: MRCPCH Part 1 B

Question: 5 of 10 / Overall score: 89%

Theme: Emergency medicine

Select the most appropriate emergency treatment for the following children: A 4-year-old child is rescued from a house fire. She is admitted tachypnoeic and tacchycardic. She has soot in her nostrils.

Correct

This relates to burns. Most deaths following house fires occur

secondary to smoke inhalation. Amongst the indicators of

inhaled smoke injury is deposits around the mouth and nose.

Oedema follows thermal injury and therefore any suspicion of

airway compromise should result in endotracheal intubation.

A 3 month old baby is admitted with a history of poor feeding. On arrival he has a pulse rate of 220 beats per minute.

Incorrect - The correct answer is Diving reflex

Supraventricular tachycardia is the diagnosis. Vagal stimulation

is the treatment of choice and the diving reflex is the simple

procedure elicited by submerging the baby's face in to ice or

placing an ice bag over the face. The diving reflex increases

vagal tone, slows AV conduction interrupting the tachycardia.

A 13-year-old boy is admitted with meningococcaemia. He is in shock. Peripheral cannulation is difficult.

Correct

A Adenosine

B Adrenaline

C Atropine

D DC shock

E Dobutamine

F Diving reflex

G Endotracheal intubation

H Intraosseous line

I IV Morphine

J Naloxone

66Endotracheal intubation

66DC shock

66Intraosseous line

 

Stats

 

Comments

 

Web links

Question statistics

Average score

78.40%

Times answered

213

Average user rating

4

 

My account Sign out

Page 31: cadiology 1.pdf

© 2010 BMJ Publishing Group Ltd . All rights reserved . Terms and

conditions . Privacy policy . A A A

This relates to a child in shock. In many life-threatening

conditions venous cannulation is difficult. It is important to

obtain vascular access very quickly and therefore intraosseous

infusion is recommended.

Rate this question

Next question

Page 32: cadiology 1.pdf

NOTE: Many features on this website require Javascript and cookies. You can enable both via your browser's preference settings.

Dashboard

Questions

Scores

Community

Help

Shop

Work smart Work hard Mock exams Revision advice

 

Work smart: MRCPCH Part 1 B

Question: 10 of 10 / Overall score: 81%

Theme: Emergency treatments

Select the most appropriate emergency treatment for the

following children.

A 5-year-old boy is brought to the hospital with 15% scalds to his chest.

Incorrect - The correct answer is IV morphine

This relates to a child with serious burns. Children who have

been burnt are in severe pain and therefore IV morphine is the

analgesic of choice.

A 14-year-old girl with a history of previous overdoses is admitted to the Emergency Department apnoeic and unconsciousness. ECG shows ventricular fibrillation. CPR is commenced.

Correct

This case describes a child in ventricular fibrillation. This is

uncommon in childhood although may occur as a result of

tricyclic antidepressant overdose and hypothermia. If the

arrest is witnessed a precordial thump is carried out otherwise

electrical de-fibrillation at 4 joules per kilogram.

A term baby is born in poor condition. Apgar scores 3 at 1 minute and 5 at 5 minutes. CPR is commenced. At 10 minutes he remains bradycardic.

Incorrect - The correct answer is Adrenaline

A Adenosine

B Adrenaline

C Atropine

D DC shock

E Dobutamine

F Diving reflex

G Endotracheal intubation

H Intraosseous line

I IV morphine

J Naloxone

66Endotracheal intubation

66DC shock

66Atropine

 

Stats

 

Comments

 

Web links

Question statistics

Average score

68.62%

Times answered

222

Average user rating

4

 

My account Sign out

Page 33: cadiology 1.pdf

© 2010 BMJ Publishing Group Ltd . All rights reserved . Terms and

conditions . Privacy policy . A A A

This case describes a baby born in poor condition. A

bradycardia in an unstable newborn requires oxygenation,

ventilation and cardiac compressions. IV adrenaline is

administered, as atropine is ineffective in this age group.

 

Rate this question

Finish

Page 34: cadiology 1.pdf

NOTE: Many features on this website require Javascript and cookies. You can enable both via your browser's preference settings.

Dashboard

Questions

Scores

Community

Help

Shop

Work smart Work hard Mock exams Revision advice

 

Work smart: MRCPCH Part 1 B

Question: 9 of 10 / Overall score: 74%

A 3 month old girl presents with liver enlargement of 4 cm.

She is slightly breathless and not gaining weight well. Full

term normal delivery, noted to have Down's facies,

confirmed as trisomy 21 on karyotype. Immunisations up

to date. No family or social history of note.

On examination:

She has cutaneous stigmata of Downs. Pulses are normal

in all 4 limbs. She has an active praecordium with loud S2

and pansystolic murmur loudest at the lower left sternal

border and apex. Liver is 4 cm and spleen 3 cm.

What is the most likely diagnosis?

(Please select 1 option)

The key points are the Down's syndrome, heart failure, with

loud murmurs. 50% of Down syndrome patients have structural

congenial heart disease. The commonest abnormalities is atrio-

ventricular canal defect followed by VSD. In the former (ostium

primum ASD) there is a defect in the lower part of the atrial

septum, the medial part of the tricuspid and mitral valves, and

in the upper part of the ventricular septum. Hence there are

overlapping murmurs of ASD, VSD, tricuspid and mitral

regurgitation together with heart failure. After detailed

evaluation by echocardiography, surgical repair is undertaken.

Rate this question

Temperature 36.5C

Respiratory rate 45/min (mild recession),

Pulse 140/min

nmlkj Aortic coarctation

nmlkj Atrial septal defect, primum This is the correct answer

nmlkj Atrial septal defect, secundum

nmlkj Hypoplastic left heart syndrome

nmlkji Ventricular septal defect Incorrect answer selected

Next question

 

Stats

 

Comments

 

Web links

Question statistics

Option analysis

Times answered

563

Average user rating

4

 

1

2

3

4

5

0%

18%

7%

1%

74%

My account Sign out

Page 35: cadiology 1.pdf

© 2010 BMJ Publishing Group Ltd . All rights reserved . Terms and

conditions . Privacy policy . A A A

Page 36: cadiology 1.pdf

NOTE: Many features on this website require Javascript and cookies. You can enable both via your browser's preference settings.

Dashboard

Questions

Scores

Community

Help

Shop

© 2010 BMJ Publishing Group Ltd . All rights reserved . Terms and conditions . Privacy policy . A A A

Work smart Work hard Mock exams Revision advice

 

Work smart: MRCPCH Part 1 B

Question: 8 of 10 / Overall score: 83%

An 8-year-old girl presents with a 2 week history of

headache. This has been severe, worst in the mornings,

accompanied by effortless vomiting, and has kept her off

school the past 4 days. Her school performance has been

deteriorating over the past term. Full term normal delivery

with no neonatal complications. Immunisations up to date.

There is no FH/SH of note.

On examination she is on the 50% for height and weight,

apyrexial, with pulse 70/min, respiratory rate 15/min and

blood pressure is 125/100 mmHg. Fundoscppy reveals

blurring of the disk margins bilaterally.

What is the most likely diagnosis?

(Please select 1 option)

The brief duration of headache, its severity and

characteristics, the absence of family history of migraine, and

findings of slow pulse, raised BP and papilloedema all point to

raised intracranial pressure. The most likely diagnosis is a

tumour. In childhood 2/3 are in the posterior fossa, often

presenting with ataxia or headache. 1/3 are supratentorial,

where raised intracranial pressure and decline in school

performance are commoner presentations.

Rate this question

nmlkj Cerebral abscess

nmlkji Medulloblastoma Correct

nmlkj Migraine

nmlkj Sinusitis

nmlkj Tension headache

Next question

 

Stats

 

Comments

 

Web links

Question statistics

Option analysis

Times answered

256

Average user rating

3

 

1

2

3

4

5

3%

96%

1%

0%

0%

My account Sign out

Page 37: cadiology 1.pdf
Page 38: cadiology 1.pdf

NOTE: Many features on this website require Javascript and cookies. You can enable both via your browser's preference settings.

Dashboard

Questions

Scores

Community

Help

Shop

Work smart Work hard Mock exams Revision advice

 

Work smart: MRCPCH Part 1 B

Question: 8 of 10 / Overall score: 92%

A 4-month-old boy presents with heart murmur,

breathlessness and failure to thrive.

He has always breathed a little fast, but this has gradually

worsened. He has been falling through the centiles. He was

born at 40+2/40 weighing 3.22 kg and there were no

neonatal problems. Immunisations are up to date. There is

no family or social history of note.

On examination the temperature is 36.9°C, respiratory rate

40/min (minimal recession) and pulse is 120/min. He is

below the 3% for weight and on the 50% for head

circumference.

He has an active praecordium, loud P2 and a 3/6

pansystolic murmur maximal at the lower left sternal edge,

but heard all over the praecordium.

The liver is 3 cm.

What is the most likely diagnosis?

(Please select 1 option)

This history suggests heart failure, worsening over the first

few weeks of life, with a pansystolic murmur.

This suggests a VSD causing haemodynamically significant

left-to-right shunt. The shunt volume usually increases in

the first few weeks as the pulmonary vascular resistance

drops. The murmur goes from ejection systolic in the first

few days, gradually becoming pansystolic. Diuretics are

often needed.

Some VSDs (particularly of the muscular septum) close

spontaneously. Others (especially of the membranous

septum) require surgical closure.

Antibiotic prophylaxis is essential to reduce the risk of

nmlkj Atrial septal defect, primum

nmlkj Atrial septal defect, secundum

nmlkj Mitral regurgitation

nmlkj Tricuspid atresia

nmlkji Ventricular septal defect (VSD) Correct

 

Stats

 

Comments

 

Web links

Question statistics

Option analysis

Times answered

537

Average user rating

4

 

1

2

3

4

5

8%

5%

4%

4%

79%

My account Sign out

Page 39: cadiology 1.pdf

© 2010 BMJ Publishing Group Ltd . All rights reserved . Terms and

conditions . Privacy policy . A A A

endocarditis.

Rate this question

Next question

Page 40: cadiology 1.pdf

NOTE: Many features on this website require Javascript and cookies. You can enable both via your browser's preference settings.

Dashboard

Questions

Scores

Community

Help

Shop

© 2010 BMJ Publishing Group Ltd . All rights reserved . Terms and conditions . Privacy policy . A A A

Work smart Work hard Mock exams Revision advice

 

Work smart: MRCPCH Part 1 B

Question: 6 of 10 / Overall score: 78%

A 14-year-old boy falls awkwardly while swinging from a

rope in an adventure playground. He lands head first,

causing violent flexion of his neck. An ambulance is called.

He has previously been fit and well. Full term normal

delivery, no neonatal problems. Immunisations up to date.

No family or social history of note.

On examination the temperature is 36.5°C. He has been

intubated and is being bag ventilated in 100% Oxygen.

Heart rate is 60/min and pulse is thready, though the

peripheries are warm and pink. Blood pressure is 75 mmHg

systolic by dynamap. He is paralysed from the neck down,

and has a GCS of 8.

What is the most mechanism of shock?

(Please select 1 option)

The history suggests spinal cord transection causing

neurogenic shock. Severe head injury or cord transection can

prevent autonomic control of blood vessels, leading to fluid

loss into the tissues (distributive shock). Unusually for shock

the pulse is slow and the peripheries warm. Fluid replacement

and treatment of the underlying cause (if possible) is

indicated.

Rate this question

nmlkj Anaemic

nmlkj Cardiogenic

nmlkji Distributive Correct

nmlkj Hypovolaemic

nmlkj Septic

Next question

 

Stats

 

Comments

 

Web links

Question statistics

Option analysis

Times answered

268

Average user rating

3

 

1

2

3

4

5

1%

28%

62%

9%

0%

My account Sign out

Page 41: cadiology 1.pdf
Page 42: cadiology 1.pdf

NOTE: Many features on this website require Javascript and cookies. You can enable both via your browser's preference settings.

Dashboard

Questions

Scores

Community

Help

Shop

Work smart Work hard Mock exams Revision advice

 

Work smart: MRCPCH Part 1 B

Question: 5 of 10 / Overall score: 88%

A 2-week-old girl presents with a history of poor feeding,

worse over the last six hours. 38+3/40, 2.95 kg, no

problems at birth. First pregnancy of healthy Caucasian

mother. No FH/SH of note. No drugs nor immunisations.

On examination temperature is 35.4°C (tympanic), RR

60/min, HR 160/min on monitor (all pulses impalpable).

Cold mottled peripheries, capillary refill time six seconds.

Oxygen saturations 45% in air. Gallop rhythm, 4 cm

hepatomegaly, no spleen. No bruising or rash.

What is the most likely diagnosis?

(Please select 1 option)

The history is of shock, hepatomegaly and hypoxia in a 2-

week-old child.

The most likely diagnosis is duct-dependent congenital

heart disease. Left heart lesions causing this presentation

include:

aortic coarctation/hypoplastic arch

truncus arteriosus

critical aortic stenosis

hypoplastic left heart syndrome.

Right-sided lesions include:

transposition

severe Fallot's

pulmonary atresia +/- VSD.

After intubation and ventilation the key step is to open the

duct using prostaglandin E2 infusion.

Acidosis may require separate correction.

nmlkj Congenital adrenal hyperplasia

nmlkj Escherichia coli septicaemia

nmlkj Galactosaemia

nmlkj Group B streptococcal infection

nmlkji Hypoplastic left heart syndrome Correct

 

Stats

 

Comments

 

Web links

Question statistics

Option analysis

Times answered

263

Average user rating

4

 

1

2

3

4

5

2%

4%

5%

16%

72%

My account Sign out

Page 43: cadiology 1.pdf

© 2010 BMJ Publishing Group Ltd . All rights reserved . Terms and

conditions . Privacy policy . A A A

Urgent transfer to a cardiac centre should then follow so a

specific diagnosis can be made.

Rate this question

Next question

Page 44: cadiology 1.pdf

NOTE: Many features on this website require Javascript and cookies. You can enable both via your browser's preference settings.

Dashboard

Questions

Scores

Community

Help

Shop

Work smart Work hard Mock exams Revision advice

 

Work smart: MRCPCH Part 1 B

Question: 3 of 10 / Overall score: 78%

Theme: Syncope.

For each scenario choose the most likely diagnosis:

An 8-year-old girl presents with loss of consciousness and occasional awareness of heartbeat. She has been deaf from birth.

Correct

The 8-year-old girl has deafness and palpitations, followed by

syncope. The most likely diagnosis is the Jervell-Lange-Neilsen

variant of long QT syndrome. The milder form is the Romano-

Ward syndrome.

A 13-year-old girl was found unconscious one Sunday morning whilst in bed. She had wet herself and took 30 minutes to recover completely.

Correct

The 13-year-old girl has had a generalised seizure as

suggested by incontinence and the prolonged recovery.

An 18 month old boy is referred with loss of consciousness on 6 occasions. Each was preceded by a tantrum.

Incorrect - The correct answer is Breath-holding, blue

The 18 month old boy has blue breath-holding episodes. These

should be distinguished from white breath-holding, which is an

extreme vagal response resulting in transient asystole.

A Arrhythmia

B Breath-holding, blue

C Breath-holding, white

D Hypertrophic cardiomyopathy

E Hypoglycaemia

F Hypotention, vasovagal

G Hypotension, orthostatic

H Seizure

I Tumour, brain

66Arrhythmia

66Seizure

66Tumour, brain

Next question

 

Stats

 

Comments

 

Web links

Question statistics

Average score

74.33%

Times answered

274

Average user rating

4

 

My account Sign out

Page 45: cadiology 1.pdf

© 2010 BMJ Publishing Group Ltd . All rights reserved . Terms and

conditions . Privacy policy . A A A

Rate this question

Page 46: cadiology 1.pdf

NOTE: Many features on this website require Javascript and cookies. You can enable both via your browser's preference settings.

Dashboard

Questions

Scores

Community

Help

Shop

© 2010 BMJ Publishing Group Ltd . All rights reserved . Terms and conditions . Privacy policy . A A A

Work smart Work hard Mock exams Revision advice

 

Work smart: MRCPCH Part 1 B

Question: 1 of 10 / Overall score: 100%

An 16-year-old female is found to be hypertensive at 26

weeks of her first pregnancy.

What is the most appropriate management?

(Please select 1 option)

This patient has Pre-eclampsia, which is pregnancy induced

hypertension with proteinuria with or without oedema; a

multisystem disorder originating in the placenta. As BP is

>140/90 with associated proteinuria she needs admission for

observation and monitoring of BP, biochemistry, daily weight,

fluid balance and monitoring of urinary protein. Methyldopa

would be the agent of choice to control hypertension. ACEi are

contraindicated in pregnancy due to teratogenicity.

Rate this question

Blood pressure 144/92 mmHg

Dipstick urine +++ proteinuria

Serum creatinine 80 µmol/L

24 hour urine collection 1.2 g of protein/d

nmlkj Atenolol

nmlkji Methyldopa Correct

nmlkj Nifedipine

nmlkj Ramipril

nmlkj Re-measure blood pressure in one week

Next question

 

Stats

 

Comments

 

Web links

Question statistics

Option analysis

Times answered

322

Average user rating

4

 

1

2

3

4

5

9%

48%

25%

3%

15%

My account Sign out

Page 47: cadiology 1.pdf

NOTE: Many features on this website require Javascript and cookies. You can enable both via your browser's preference settings.

Dashboard

Questions

Scores

Community

Help

Shop

© 2010 BMJ Publishing Group Ltd . All rights reserved . Terms and conditions . Privacy policy . A A A

Work smart Work hard Mock exams Revision advice

 

Work smart: MRCPCH Part 1 B

Question: 3 of 10 / Overall score: 80%

A 17-year-old boy whose brother had hypertrophic

cardiomyopathy was referred for a cardiological

assessment. His echocardiogram confirmed the condition.

Which one of the following echocardiographic features is

the most important risk factor for sudden cardiac death?

(Please select 1 option)

In hypertrophic obstructive cardiomyopathy the cause of

death is usually ventricular tachycardia or ventricular

fibrillation and therefore the thicker the muscle the more

abnormal the cardiac architecture and the higher the risk of

arrhythmia and sudden death.

Rate this question

nmlkj A gradient of 10 mmHg across the left ventricular outflow tract

nmlkji Significant thickening of the interventricular septum Correct

nmlkj An enlarged left atrium

nmlkj Systolic anterior motion of the mitral valve

nmlkj The presence of mitral regurgitation

Next question

 

Stats

 

Comments

 

Web links

Question statistics

Option analysis

Times answered

164

Average user rating

5

 

1

2

3

4

5

22%

54%

10%

8%

6%

My account Sign out

Page 48: cadiology 1.pdf

NOTE: Many features on this website require Javascript and cookies. You can enable both via your browser's preference settings.

Dashboard

Questions

Scores

Community

Help

Shop

© 2010 BMJ Publishing Group Ltd . All rights reserved . Terms and conditions . Privacy policy . A A A

Work smart Work hard Mock exams Revision advice

 

Work smart: MRCPCH Part 1 B

Question: 9 of 10 / Overall score: 55%

The following are true of cardiac transplantation:

True / False

a-This is true of kidney transplants but hearts go to the most

needy. b-Hypertension is a significant problem related to use

of cyclosporin and denervation of the heart. c-The heart is

denervated. d-The pathology is different being more diffuse

and symmetrical. e-For most centres with conventional

selection criteria.

Transplantation

Rate this question

nmlkji nmlkj the donor heart is provided to the recipient with the best HLA match Incorrect answer selected

nmlkji nmlkj long term hypotension is a problem Incorrect answer selected

nmlkji nmlkj recipients suffer less angina but accelerated coronary artery disease This is the correct answer

nmlkj nmlkji the histology of atheroma in the transplanted heart is the same as that of typical atherosclerosis Correct

nmlkji nmlkj 75% 5 year survival Correct

Next question

 

Stats

 

Comments

 

Web links

Question statistics

Average score

48.92%

Times answered

148

Average user rating

5

 

My account Sign out

Page 49: cadiology 1.pdf

NOTE: Many features on this website require Javascript and cookies. You can enable both via your browser's preference settings.

Dashboard

Questions

Scores

Community

Help

Shop

© 2010 BMJ Publishing Group Ltd . All rights reserved . Terms and conditions . Privacy policy . A A A

Work smart Work hard Mock exams Revision advice

 

Work smart: MRCPCH Part 1 B

Question: 8 of 10 / Overall score: 57%

Cerebral abscess in the absence of endocarditis is a recognised complication of:

True / False

This is paradoxical embolization seen in right to left shunts. It

is not associated with left to right shunts except if

Eisenmenger's supervenes.

Rate this question

nmlkji nmlkj tetralogy of Fallot Correct

nmlkj nmlkji persistent ductus arteriosus Correct

nmlkji nmlkj tricuspid artresia Correct

nmlkji nmlkj transposition of the great arteries Correct

nmlkj nmlkji atrial septal defect of the ostium primum type Correct

Next question

 

Stats

 

Comments

 

Web links

Question statistics

Average score

55.43%

Times answered

162

Average user rating

1

 

My account Sign out

Page 50: cadiology 1.pdf

NOTE: Many features on this website require Javascript and cookies. You can enable both via your browser's preference settings.

Dashboard

Questions

Scores

Community

Help

Shop

© 2010 BMJ Publishing Group Ltd . All rights reserved . Terms and conditions . Privacy policy . A A A

Work smart Work hard Mock exams Revision advice

 

Work smart: MRCPCH Part 1 B

Question: 7 of 10 / Overall score: 50%

A short PR interval on the ECG is associated with:

True / False

a-Commonest change is large R/S ratio in V1, but short PR

interval can also occur. b-Associated with a prolonged PR

interval which may manifest before the overt clinical

manifestations of the disease. (Am J Med 1978(61), 452). c-In

a quarter of cases (JACC 1986(7), 1370-8). d-Long PR. e-In

some cases HOCM is associated with a short PR. Commoner

causes are WPW and Lown-Ganong-Levine syndromes.

ECG changes in Duchenne

Rate this question

nmlkj nmlkji Duchenne muscular dystrophy Incorrect answer selected

nmlkj nmlkji dystrophia myotonica Correct

nmlkji nmlkj Friedrich's ataxia Correct

nmlkji nmlkj rheumatic carditis Incorrect answer selected

nmlkj nmlkji hypertrophic cardiomyopathy Incorrect answer selected

Next question

 

Stats

 

Comments

 

Web links

Question statistics

Average score

50.53%

Times answered

169

Average user rating

3

 

My account Sign out

Page 51: cadiology 1.pdf

NOTE: Many features on this website require Javascript and cookies. You can enable both via your browser's preference settings.

Dashboard

Questions

Scores

Community

Help

Shop

© 2010 BMJ Publishing Group Ltd . All rights reserved . Terms and conditions . Privacy policy . A A A

Work smart Work hard Mock exams Revision advice

 

Work smart: MRCPCH Part 1 B

Question: 5 of 10 / Overall score: 56%

The following are recognised causes of pericardial disease:

True / False

a-And Coxsackie B (the commonest viral cause), ECHO, rubella,

mumps and influenza viruses. b-An autosomal recessive

condition affecting MUscle LIver BRain and EYes and includes

pericardial fibrosis. c-A small effusion accompanies almost all

cases of acute rheumatic fever. d-This affects the

endomyocardium causing fibrosis but may also cause a

pericarditis. e-Aortic regurgitation and AV block are recognised

consequences of ankylosing spondylitis.

Rate this question

nmlkji nmlkj Hepatitis B virus Correct

nmlkj nmlkji Mulibrey nanism Incorrect answer selected

nmlkji nmlkj rheumatic fever Correct

nmlkj nmlkji hypereosinophilic syndrome Incorrect answer selected

nmlkji nmlkj ankylosing spondylitis Incorrect answer selected

Next question

 

Stats

 

Comments

 

Web links

Question statistics

Average score

58.44%

Times answered

154

Average user rating

3

 

My account Sign out

Page 52: cadiology 1.pdf

NOTE: Many features on this website require Javascript and cookies. You can enable both via your browser's preference settings.

Dashboard

Questions

Scores

Community

Help

Shop

Work smart Work hard Mock exams Revision advice

 

Work smart: MRCPCH Part 1 B

Question: 2 of 10 / Overall score: 70%

Theme: Syndromes associated with congenital heart disease

For each of the cardiac lesions described below, choose the

most commonly associated syndrome from the above list

of options. Each option may be used once or not at all.

Supravalvular aortic stenosis

Incorrect - The correct answer is William's syndrome

William's syndrome is characterised by

short stature

characteristic facies

supravalvular aortic stenosis

mild to moderate learning difficulties

transient neonatal hypercalcaemia.

Coarctation of aorta

Correct

This is characterised by

45,X genotype

ovarian dysgenesis leading to infertility

short stature

webbing of the neck

wide carrying angles

wide spaced nipples.

A Charcot-Marie-Tooth syndrome

B Down syndrome

C Fragile X syndrome

D Marfan's syndrome

E Noonan syndrome

F Sturge-Weber syndrome

G Tetrology of Fallot

H Tourette syndrome

I Turner's syndrome

J William's syndrome

66Sturge-Weber syndrome

66Turner's syndrome

 

Stats

 

Comments

 

Web links

Question statistics

Average score

78.21%

Times answered

179

Average user rating

5

 

My account Sign out

Page 53: cadiology 1.pdf

However they have normal intellectual development.

Dilatation of aortic root/aortic regurgitation

Correct

This is an autosomal dominant disorder. The clinical

features are

tall stature

arachnodactyly

high arched plate

increase in length of the lower segment of the body compared to the upper segment.

The cardiac manifestations include

dilated aortic root

aortic incompetence

mitral valve prolapse

mitral incompetence.

Cardiac cushion defects (leading to ASD, VSD)

Incorrect - The correct answer is Down syndrome

Features include

characteristic facies

hypotonia

severe learning difficulties

small stature.

About 40% of patients have cardiac anomalies, mainly

endocardial cushion defects leading to ASD and VSD.

Infundibular pulmonary stenosis

Incorrect - The correct answer is Tetrology of Fallot

This is a cyanotic heart disease and the cardinal features

include

infundibular pulmonary stenosis

VSD

right ventricular hypertrophy

over-riding of the aorta.

Charcot-Marie-Tooth syndrome: autosomal dominant

peroneal muscular dystrophy.

Fragile X syndrome: moderate learning difficulty,

66Marfan's syndrome

66William's syndrome

66Noonan syndrome

Page 54: cadiology 1.pdf

© 2010 BMJ Publishing Group Ltd . All rights reserved . Terms and

conditions . Privacy policy . A A A

macrocephaly, characteristic facies (long face, large ears,

prominent mandible and forehead).

Noonan syndrome: facies, mild learning difficulties, short

webbed neck, short stature and congenital heart disease

(pulmonary valvular stenosis, ASD).

Sturge-Weber syndrome: haemangiomas in the distribution

of trigeminal nerve and in the brain. Sometimes can have

intractable epilepsy.

Tourette syndrome: tics, compulsive utterances of obscene

words (coprolalia).

Rate this question

Next question

Page 55: cadiology 1.pdf

NOTE: Many features on this website require Javascript and cookies. You can enable both via your browser's preference settings.

Dashboard

Questions

Scores

Community

Help

Shop

© 2010 BMJ Publishing Group Ltd . All rights reserved . Terms and conditions . Privacy policy . A A A

Work smart Work hard Mock exams Revision advice

 

Work smart: MRCPCH Part 1 B

Question: 2 of 40 / Overall score: 30%

The following regarding Atrial Natriuretic Peptide are correct

True / False

ANP is a 28 aa peptide that is synthesised and released by

atrial myocytes in response to distension, angiotensin II and

endothelin. It causes salt and hence water losses, is a

vasodilator (through possible direct and indirect mechanisms)

and inhibits aldosterone release. Such actions reduce blood

volume and cardiac output. ANP is degraded by neutral

endopeptidase.

Rate this question

nmlkji nmlkj It is secreted in response to right atrial distension Correct

nmlkji nmlkj It inhibits aldosterone release Correct

nmlkji nmlkj It is a direct inotrope Incorrect answer selected

nmlkj nmlkji It is a direct vasodilator Incorrect answer selected

nmlkji nmlkj It is inactivated by endopeptidase Correct

Next question

 

Stats

 

Comments

 

Web links

Question statistics

Average score

65.86%

Times answered

215

Average user rating

3

 

My account Sign out

Page 56: cadiology 1.pdf

NOTE: Many features on this website require Javascript and cookies. You can enable both via your browser's preference settings.

Dashboard

Questions

Scores

Community

Help

Shop

© 2010 BMJ Publishing Group Ltd . All rights reserved . Terms and conditions . Privacy policy . A A A

Work smart Work hard Mock exams Revision advice

 

Work smart: MRCPCH Part 1 B

Question: 1 of 40 / Overall score: 0%

Which one of the following is a recognised feature of abetalipoproteinaemia?

(Please select 1 option)

Acanthocytes are seen in abetalipoproteinaemia.

Retinitis pigmentosa is seen in abetalipoproteinaemia.

Neurodegenerative changes are seen such as ataxia but IQ is

normal.

Rate this question

nmlkji a high serum cholesterol Incorrect answer selected

nmlkj palmar xanthomas

nmlkj advanced atherosclerotic vascular disease

nmlkj abnormal red blood cell morphology This is the correct answer

nmlkj Severe mental retardation

Next question

 

Stats

 

Comments

 

Web links

Question statistics

Option analysis

Times answered

190

Average user rating

3

 

1

2

3

4

5

12%

11%

12%

49%

16%

My account Sign out

Page 57: cadiology 1.pdf

NOTE: Many features on this website require Javascript and cookies. You can enable both via your browser's preference settings.

Dashboard

Questions

Scores

Community

Help

Shop

© 2010 BMJ Publishing Group Ltd . All rights reserved . Terms and conditions . Privacy policy . A A A

Work smart Work hard Mock exams Revision advice

 

Work smart: MRCPCH Part 1 B

Question: 10 of 10 / Overall score: 65%

A post-marketing surveillance study of a new heart failure

therapy to the market was carried out on 10,000 subjects

who had completed clinical trials. Which one of the following

most accurately reflects the information generated from

such a study?

(Please select 1 option)

Post-marketing surveillance/observational studies (phase IV

studies) generally are designed to assess the potential side

effects of new drugs but under everyday conditions and

with a minimum of intervention. In contrast to the

randomized controlled trials, PMS typically include patients

from more extreme age groups, patients with comorbidity

or other risk factors. In order to cover a wide spectrum of

patients and to observe rare events with sufficiently high

probability, PMS enroll a large number of patients, typically

several thousands. Comparative efficacy has already been

undertaken in Phase III studies (RCTs) but can also be

undertaken as part of specific RCT studies later in the

drugs development and potency usually in phase I and II

studies.

Rate this question

nmlkj Adverse events profile This is the correct answer

nmlkj Cost benefit analysis

nmlkji Cost effectiveness Incorrect answer selected

nmlkj Comparative therapeutic efficacy

nmlkj Drug potency

Finish

 

Stats

 

Comments

 

Web links

Question statistics

Option analysis

Times answered

204

Average user rating

4

 

1

2

3

4

5

39%

10%

8%

31%

13%

My account Sign out

Page 58: cadiology 1.pdf

NOTE: Many features on this website require Javascript and cookies. You can enable both via your browser's preference settings.

Dashboard

Questions

Scores

Community

Help

Shop

Work smart Work hard Mock exams Revision advice

 

Work smart: MRCPCH Part 1 B

Question: 6 of 10 / Overall score: 52%

Theme: Congenital heart disease

Which of the above is the most likely diagnosis in the

following cases?

A 7-week-old infant presents with breathlessness on feeding and failure to thrive. On examination his femoral pulses are difficult to feel but present. Chest radiograph shows cardiomegaly and increased vascular markings.

Incorrect - The correct answer is Coarctation of the aorta

Absent or weak femoral pulses suggest coarctation.

Remember association with Turner's syndrome in females.

A 13-year-old girl is referred for evaluation of her short stature. She is pre-pubertal. On auscultation she has an ejection systolic murmur in the second and third left intercostal spaces radiating to the back, but is asymptomatic.

Incorrect - The correct answer is Pulmonary valve stenosis

The murmur describes pulmonary stenosis which could also

be a left peripheral pulmonary stenosis. She is short and

has delayed puberty, and coupled with the cardiac findings,

would suggest Noonan's syndrome.

An infant is seen for his 6-week-check and found to have a loud ejection systolic murmur in the third left intercostal space and a single second heart sound on examination. There is no obvious cyanosis but a suggestion of mild desaturation.

On the chest x ray there is a concavity on the left heart border

A Ostium secundum atrial septal defect

B Ventricular septal defect

C Transposition of the great arteries

D Total anomalous pulmonary venous drainage

E Atrioventricular septal defect

F Patent ductus arteriosus

G Pulmonary valve stenosis

H Coarctation of the aorta

I Tetralogy of Fallot

J Hypoplastic left heart syndrome

66Ventricular septal defect

66Coarctation of the aorta

 

Stats

 

Comments

 

Web links

Question statistics

Average score

46.67%

Times answered

180

Average user rating

4

 

My account Sign out

Page 59: cadiology 1.pdf

© 2010 BMJ Publishing Group Ltd . All rights reserved . Terms and

conditions . Privacy policy . A A A

and decreased pulmonary vascular markings.

Correct

Tetralogy of Fallot may present later than in the neonatal

period. The ejection systolic murmur is from the

infundibular stenosis. The desaturation results from the

right to left shunt across the VSD.

 

Rate this question

66Tetralogy of Fallot

Next question

Page 60: cadiology 1.pdf

NOTE: Many features on this website require Javascript and cookies. You can enable both via your browser's preference settings.

Dashboard

Questions

Scores

Community

Help

Shop

© 2010 BMJ Publishing Group Ltd . All rights reserved . Terms and conditions . Privacy policy . A A A

Work smart Work hard Mock exams Revision advice

 

Work smart: MRCPCH Part 1 B

Question: 1 of 10 / Overall score: 100%

A 16-year-old boy is admitted after a blackout at the

dentist. His mother describes how he blacked out as the

dentists began performing a filling and that he jerked his

arms a few times and was then incontinent. He awoke after

a minute or so and was oriented but nauseous.There were

no similar episodes in the past and he is totally unaware of

what happened.

Examination was normal and his ECG was normal. Which

one of the following is the most likely diagnosis?

(Please select 1 option)

Vasovagal syncope is common during dental procedures,

mainly induced by pain (as the dentist started drilling). The

fact that he recovered very quickly supports the diagnosis

of syncope. It is common to have jerking of limbs due to

brain hypoxia.

ECG is always normal. Incontinence of urine can occur, but

not biting of the tongue.

 

Rate this question

nmlkj Complex partial seizure

nmlkj Pseudoseizure

nmlkj Stokes-Adams attack

nmlkj Tonic-clonic seizure

nmlkji Vasovagal syncope Correct

Next question

 

Stats

 

Comments

 

Web links

Question statistics

Option analysis

Times answered

224

Average user rating

5

 

1

2

3

4

5

14%

4%

7%

11%

64%

My account Sign out

Page 61: cadiology 1.pdf

NOTE: Many features on this website require Javascript and cookies. You can enable both via your browser's preference settings.

Dashboard

Questions

Scores

Community

Help

Shop

© 2010 BMJ Publishing Group Ltd . All rights reserved . Terms and conditions . Privacy policy . A A A

Work smart Work hard Mock exams Revision advice

 

Work smart: MRCPCH Part 1 B

Question: 4 of 10 / Overall score: 95%

A 16-year-old male is brought to emergency admissions

with alcohol intoxication. An initial ECG reveals atrial

fibirillation but a repeat ECG after 12 hours when he has

sobered up, shows sinus rhythm. An echocariogram is

normal. What is the most appropriate management for this

patient?

(Please select 1 option)

Excessive Alcohol is a recognized cause for atrial fibirillation

and is the likely cause here as the rhythm has reverted to

sinus after 12 hours. There is also no evidence of

structural heart disease as the echocardiogram was normal.

Therefore this patient needs advice regarding moderation

of alcohol consumption and needs to be warned of the

toxic effects that alcohol can have on the heart and other

organs.

There is no indication for short term aspirin. Atenolol

provides rate control, which is not an issue.

Sotalol/amiodarone and Flecainide can be used in

paroxysmal AF. Short term Warfarin is used for 4-6 weeks

prior to elective cardioversion to protect against embolic

complications.

Rate this question

nmlkj Aspirin for 3 months

nmlkj Bisoprolol for 3 months

nmlkji Lifestyle advice Correct

nmlkj Sotalol for one month

nmlkj Warfarin for one month

Next question

 

Stats

 

Comments

 

Web links

Question statistics

Option analysis

Times answered

213

Average user rating

4

 

1

2

3

4

5

5%

0%

92%

0%

3%

My account Sign out

Page 62: cadiology 1.pdf

NOTE: Many features on this website require Javascript and cookies. You can enable both via your browser's preference settings.

Dashboard

Questions

Scores

Community

Help

Shop

© 2010 BMJ Publishing Group Ltd . All rights reserved . Terms and conditions . Privacy policy . A A A

Work smart Work hard Mock exams Revision advice

 

Work smart: MRCPCH Part 1 B

Question: 3 of 10 / Overall score: 93%

A 16-year-old male presents with acute severe asthma. On

examination his peripheral pulse volume fell during

inspiration.

Which one of the following is the most likely explanation for

this clinical sign?

(Please select 1 option)

This patient is demonstrating pulsus paradoxus. The right

heart responds directly to changes in intrathoracic

pressure, while the filling of the left heart depends on the

pulmonary vascular volume. At high respiratory rates, with

severe air flow limitation (eg acute asthma) there is an

increased and sudden negative intrathoracic pressure on

inspiration and this will enhance the normal fall in blood

pressure.

Rate this question

nmlkj The cardiac effect of high dose beta agonist bronchodilator drugs

nmlkj A falling heart rate on inspiration

nmlkj Myocardial depression due to hypoxia

nmlkj Peripheral vasodilatation

nmlkji Reduced left atrial filling pressure on inspiration Correct

Next question

 

Stats

 

Comments

 

Web links

Question statistics

Option analysis

Times answered

220

Average user rating

5

 

1

2

3

4

5

1%

14%

6%

3%

76%

My account Sign out

Page 63: cadiology 1.pdf

NOTE: Many features on this website require Javascript and cookies. You can enable both via your browser's preference settings.

Dashboard

Questions

Scores

Community

Help

Shop

Work smart Work hard Mock exams Revision advice

 

Work smart: MRCPCH Part 1 B

Question: 2 of 10 / Overall score: 90%

In disorders of cardiac conduction:

True / False

The bundle of His consists of three fascicles: - the right

bundle branch, and the anterior and posterior fascicles of

the left bundle branch. Bifascicular block describes the

combination of right bundle branch and either left anterior

or left posterior hemiblock.M

Left anterior hemiblock causes left axis deviation, and left

posterior hemiblock causes right axis deviation. A-V

conduction is maintained by the remaining functional

fascicle. Impaired conduction in the latter may be manifest

by a prolonged P-R interval. Bifascicular block may progress

to trifascicular block and may be an indication for the

insertion of a permanent cardiac pacemaker.

Right bundle branch block (RBBB) in isolation, is a normal

ECG variant. The delayed right ventricular depolarization

results in a second positive (R???) wave in the right

ventricular leads and a second negative wave in the left

ventricular leads. Diagnostic features include a widened

QRS complex and a second positive wave in V1. Other

features include deep slurred S waves in leads I, aVL, V4-6,

and S-T segment and T wave changes in leads V1-3.

Left bundle branch block (LBBB) is always pathological. In

the presence of LBBB, the diagnosis of ventricular

hypertrophy, myocardial ischaemia/infarction, and

abnormalities of the p wave, QRS complex or S-T segment

cannot be made using conventional ECG criteria.

nmlkji nmlkj right bundle branch block and left axis deviation indicate bifascicular block Correct

nmlkj nmlkji left anterior hemiblock causes right axis deviation Correct

nmlkji nmlkj left posterior hemiblock causes left axis deviation Incorrect answer selected

nmlkji nmlkj deep S waves are found in leads I and V6 in right bundle branch block Correct

nmlkji nmlkj right ventricular hypertrophy cannot be diagnosed in the presence of left bundle branch block Correct

Next question

 

Stats

 

Comments

 

Web links

Question statistics

Average score

46.33%

Times answered

218

Average user rating

4

 

My account Sign out

Page 64: cadiology 1.pdf

© 2010 BMJ Publishing Group Ltd . All rights reserved . Terms and

conditions . Privacy policy . A A A

Rate this question

Page 65: cadiology 1.pdf

NOTE: Many features on this website require Javascript and cookies. You can enable both via your browser's preference settings.

Dashboard

Questions

Scores

Community

Help

Shop

© 2010 BMJ Publishing Group Ltd . All rights reserved . Terms and conditions . Privacy policy . A A A

Work smart Work hard Mock exams Revision advice

 

Work smart: MRCPCH Part 1 B

Question: 4 of 10 / Overall score: 85%

A 3-month-old girl presents with apnoea. She had been

well that morning, but had become unsettled, crying

inconsolably and gradually more mottled.

Mother was bringing her to the emergency department

when she stopped breathing . She responded to physical

stimulation. She was born at 40+3/40 weighing 3.6kg and

there were no neonatal problems.

On examination she has a temperature of 36.3C

(tympanic), RR 30/min and HR of 240/min. Her pulse in

thready. She has a 3 cm liver and gallop rhythm.

What is the most likely diagnosis?

(Please select 1 option)

The history suggests apnoea precipitated by

tachyarrhythmia. This is most likely to be a supraventricular

tachycardia.

This can be confirmed by ECG monitoring, and is usually

successfully reverted by adenosine with digoxin

maintenance therapy.

An echocardiogram will exclude the rare possibility of an

underlying structural defect.

Rate this question

nmlkj Acute life-threatening event

nmlkj Bronchiolitis

nmlkj Gastro-oesophageal reflux

nmlkj Seizure

nmlkji Supraventricular tachycardia Correct

Next question

 

Stats

 

Comments

 

Web links

Question statistics

Option analysis

Times answered

230

Average user rating

5

 

1

2

3

4

5

20%

0%

0%

0%

80%

My account Sign out

Page 66: cadiology 1.pdf
Page 67: cadiology 1.pdf

NOTE: Many features on this website require Javascript and cookies. You can enable both via your browser's preference settings.

Dashboard

Questions

Scores

Community

Help

Shop

Work smart Work hard Mock exams Revision advice

 

Work smart: MRCPCH Part 1 B

Question: 10 of 10 / Overall score: 77%

A 4-year-old girl presents with heart murmur and

breathlessness. She was well until 7 days ago, when she

developed a nasty cold and fever. She has progressively

deteriorated. Full term normal delivery with no neonatal

complications. Immunisations up to date. There is no

FH/SH of note.

On examination:

She is apyrexial and on the 25% centile for height and

weight. She has slightly cool peripheries and a capillary refill

time of 2 seconds. She has a loud gallop rhythm and 4 cm

liver. Pulses are easily palpable in all 4 limbs. Chest X-ray

shows globular cardiac enlargement with pulmonary

plethora.

What is the most likely diagnosis?

(Please select 1 option)

The history is of viral illness followed by heart failure. The likely

diagnosis is viral myocarditis or cardiomyopathy. An

echocardiogram shows dilated chambers with poor

contractility. Treatment is with diuretics and ACE inhibitors.

Spontaneous recovery is variable, and transplant may be

required.

Rate this question

Respiratory rate 25/min

pulse 130/min

BP 80/60 mmHg

nmlkj Aortic coarctation

nmlkj Atrioventricular canal defect

nmlkj Kawasaki disease

nmlkji Myocarditis Correct

nmlkj Ventricular septal defect

Finish

 

Stats

 

Comments

 

Web links

Question statistics

Option analysis

Times answered

208

Average user rating

4

 

1

2

3

4

5

2%

4%

3%

64%

26%

My account Sign out

Page 68: cadiology 1.pdf

© 2010 BMJ Publishing Group Ltd . All rights reserved . Terms and

conditions . Privacy policy . A A A

Page 69: cadiology 1.pdf

NOTE: Many features on this website require Javascript and cookies. You can enable both via your browser's preference settings.

Dashboard

Questions

Scores

Community

Help

Shop

© 2010 BMJ Publishing Group Ltd . All rights reserved . Terms and conditions . Privacy policy . A A A

Work smart Work hard Mock exams Revision advice

 

Work smart: MRCPCH Part 1 B

Question: 6 of 10 / Overall score: 90%

A 3-year-old boy is referred with heart murmur. He has

otherwise been well, with no blueness and normal exercise

tolerance. Full term normal delivery with no neonatal

complications. Immunisations up to date. There is no

FH/SH of note.

On examination he is well and apyrexial. There are no

dysmorphic features. Respiratory rate is 17/min and pulse

100/min. He has a widely split S2 which does not vary with

breathing, and 2/6 ejection systolic murmur at the upper

left sternal edge. Liver is impalpable.

What is the most likely diagnosis?

(Please select 1 option)

The history of asymptomatic heart murmur, with fixed

splitting of the second heart sound and pulmonary flow

murmur all point to atrial septal defect (ASD) as the

diagnosis here.

This may be treatable using 'umbrella' closure via a cardiac

catheter. Occasionally surgical closure is required if very

large or associated with other heart defects.

Antibiotic prophylaxis is essential to reduce the risk of

endocarditis, although the risk is lower than for 'high-flow'

lesions.

Rate this question

nmlkj Aortic coarctation

nmlkj Atrial septal defect, primum

nmlkji Atrial septal defect, secundum Correct

nmlkj Innocent murmur

nmlkj Pulmonary stenosis

Next question

 

Stats

 

Comments

 

Web links

Question statistics

Option analysis

Times answered

222

Average user rating

3

 

1

2

3

4

5

1%

18%

55%

3%

24%

My account Sign out

Page 70: cadiology 1.pdf
Page 71: cadiology 1.pdf

NOTE: Many features on this website require Javascript and cookies. You can enable both via your browser's preference settings.

Dashboard

Questions

Scores

Community

Help

Shop

© 2010 BMJ Publishing Group Ltd . All rights reserved . Terms and conditions . Privacy policy . A A A

Work smart Work hard Mock exams Revision advice

 

Work smart: MRCPCH Part 1 B

Question: 7 of 10 / Overall score: 81%

A 3-year-old girl is referred with incidentally-discovered

heart murmur. She has had no blueness nor breathless.

Full term normal delivery with no neonatal complications.

Immunisations up to date. There is no FH/SH of note.

On examination she is apyrexial, well and well grown with

no dysmorphic features. Respiratory rate is 20/min and

pulse is 100/min. Heart sounds are normal. She has a 1-

2/6 murmur best heard under the right clavicle, which

changes with posture. The liver is impalpable.

What is the most likely diagnosis?

(Please select 1 option)

The history is of asymptomatic heart murmur which is soft and

varies with posture. This is most likely an Innocent Murmur, in

this case a venous hum caused by the rumble of blood through

the great veins of the neck. It requires explanation and

reassurance.

Rate this question

nmlkj Aortic stenosis

nmlkj Atrial septal defect, secundum

nmlkj Atrioventricular canal defect

nmlkji Innocent murmur Correct

nmlkj Pulmonary stenosis

Next question

 

Stats

 

Comments

 

Web links

Question statistics

Option analysis

Times answered

223

Average user rating

4

 

1

2

3

4

5

5%

3%

1%

89%

2%

My account Sign out

Page 72: cadiology 1.pdf

NOTE: Many features on this website require Javascript and cookies. You can enable both via your browser's preference settings.

Dashboard

Questions

Scores

Community

Help

Shop

© 2010 BMJ Publishing Group Ltd . All rights reserved . Terms and conditions . Privacy policy . A A A

Work smart Work hard Mock exams Revision advice

 

Work smart: MRCPCH Part 1 B

Question: 10 of 10 / Overall score: 59%

An 11-day-old baby presents with poor feeding and

breathlessness. She had been born at 37/40 weighing 2.7

kg by elective caesarian section. She has never fed well,

and had deteriorated markedly on the day of admission.

On examination she is responding to pain, mottled and had

a tympanic temperature of 34.6°C. Her heart rate is

130/min with impalpable pulses and gallop rhythm. Her

respiratory rate is 40/min with marked recession. She has a

4 cm liver. Her saturations and blood pressure are

unrecordable, but she has obvious central cyanosis.

What is the most likely mechanism of shock?

(Please select 1 option)

The picture suggests duct-dependent congenital heart

disease, with progressive worsening of symptoms as the

duct closed. Typical conditions include

aortic coarctation

critical aortic stenosis

truncus arteriosus

hypoplastic left heart syndrome.

Rate this question

nmlkj Anaemic

nmlkji Cardiogenic Correct

nmlkj Distributive

nmlkj Hypovolaemic

nmlkj Septic

Finish

 

Stats

 

Comments

 

Web links

Question statistics

Option analysis

Times answered

260

Average user rating

4

 

1

2

3

4

5

1%

89%

1%

5%

4%

My account Sign out

Page 73: cadiology 1.pdf
Page 74: cadiology 1.pdf

NOTE: Many features on this website require Javascript and cookies. You can enable both via your browser's preference settings.

Dashboard

Questions

Scores

Community

Help

Shop

Work smart Work hard Mock exams Revision advice

 

Work smart: MRCPCH Part 1 B

Question: 5 of 10 / Overall score: 73%

Theme: Chest pain in children.

For each scenario choose the most likely diagnosis:

A 13-year-old girl presents with chest pain. On examination there is pain and tenderness of the 8th right costochondral cartilage.

Incorrect - The correct answer is Costochondritis

The 13-year-old girl has localised chest pain, most likely due to

costochondritis. A history of preceding viral infection or trauma

should be sought.

A 12-year-old boy presents with chest pain. He has previously fainted during a football game. On examination he has a 2/6 ejection murmur at the left sternal edge.

Incorrect - The correct answer is Hypertrophic cardiomyopathy

The 12-year-old boy has a history of collapse on exercise

together with a heart murmur would suggest a possible

diagnosis of HOCM. Mitral valve prolapse is less likely as the

murmur is described as ejection systolic in nature. A family

history of sudden death may also be present.

A 14-year-old girl presents with chest pain and dizziness. She looks breathless, anxious, and complains of tingling in her fingers.

Correct

The 14-year-old girl is hyperventilating. The finger

paraesthesiae amy represent reduced ionised calcium

associated with a respiratory alkalosis. The symptoms should

A Asthma

B Costochondritis

C Herpes zoster

D Hypertrophic cardiomyopathy

E Hyperventilation

F Mitral valve prolapse

G Musculoskeletal pain

H Pericarditis

I Psychogenic

66Pericarditis

66Musculoskeletal pain

66Hyperventilation

 

Stats

 

Comments

 

Web links

Question statistics

Average score

75.15%

Times answered

279

Average user rating

5

 

My account Sign out

Page 75: cadiology 1.pdf

© 2010 BMJ Publishing Group Ltd . All rights reserved . Terms and

conditions . Privacy policy . A A A

resolve with re-breathing into a paper bag.

Rate this question

Next question

Page 76: cadiology 1.pdf

NOTE: Many features on this website require Javascript and cookies. You can enable both via your browser's preference settings.

Dashboard

Questions

Scores

Community

Help

Shop

Work smart Work hard Mock exams Revision advice

 

Work smart: MRCPCH Part 1 B

Question: 4 of 10 / Overall score: 83%

Theme: Palpitations in children.

For each scenario choose the most likely diagnosis:

A 14-year-old girl complains of awareness of heart beat. She began menstruating at 12 years, and has heavy periods. She appears pale.

Correct

The 14-year-old girl is anaemic due to blood loss, resulting in

palpitations

A 10-year-old girl complains of awareness of heart beat. She has sweaty palms and a heart rate of 100/min, with a 2/6 ejection systolic murmur in the pulmonary area.

Correct

The 10-year-old girl has a resting tachycardia and sweating,

suggesting hyperthyroidism. Careful examination should be

made for goitre, bruit and eye signs. The murmur is caused by

increased flow across the pulmonary valve due to

hyperdynamic circulation.

A 9-year-old boy presents with awareness of heart beat and chest discomfort. Episodes last about 20-30 minutes. He drinks 4 cups of coffee per day.

Correct

The 9-year-old boy is probably having episodes of

tachyarrhythmia, probably SVT. This may be precipitated by

A Anaemia

B Cardiomyopathy

C Hyperthyroidism

D Myocarditis

E Panic attacks

F Prolonged QT syndrome

G Sinus arrhythmia

H Sinus tachycardia

I Supraventricular tachycardia

66Anaemia

66Hyperthyroidism

66Supraventricular tachycardia

 

Stats

 

Comments

 

Web links

Question statistics

Average score

62.47%

Times answered

262

Average user rating

3

 

My account Sign out

Page 77: cadiology 1.pdf

© 2010 BMJ Publishing Group Ltd . All rights reserved . Terms and

conditions . Privacy policy . A A A

caffeine (coffee, tea, coke) or pseudoephedrine (cold cures).

Caffeine itself may give sinus tachycardia. Asking the patient

to tap out the rate of heartbeat can be helpful.

Rate this question

Next question

Page 78: cadiology 1.pdf

NOTE: Many features on this website require Javascript and cookies. You can enable both via your browser's preference settings.

Dashboard

Questions

Scores

Community

Help

Shop

Work smart Work hard Mock exams Revision advice

 

Work smart: MRCPCH Part 1 B

Question: 2 of 10 / Overall score: 83%

Theme: Apnoea

For each scenario choose the most specific diagnostic investigation:

A 5 day old girl presents to casualty following a brief apnoeic episode. On examination she is apyrexial, sweating, has a high-pitched cry and is difficult to settle.

Incorrect - The correct answer is Drug screen on mother

The 5day old presents with apnoea in the absence of fever or

structural problem. The other symptoms point towards drug

withdrawal, so a drug screen will be diagnostic.

A 4 month old girl presents with a 12 hour history of poor feeding and fever. She is rushed to hospital after she stops breathing. On examination she has an Oxygen saturation of 94%, and responds to pain. Serum glucose is 3.4 mmol/l.

Correct

The 4 month old girl presents with a brief history of fever

followed by apnoea. This suggests an infective cause. The

decreased level of consciousness mean that LP should not be

done yet. Blood cultures are likely to be the most helpful test.

A 5 month old boy has a 2 day history of coryza and poor feeding. On the day of admission he has a brief period of apnoea and is rushed to hospital. On examination he has 2 cm hepatomegaly and a blood glucose of 0.8 mmol/l.

A Blood cultures

B CSF culture

C CT scan head

D Drug screen on mother

E EEG

F Nasopharyngeal aspirate for viruses

G Nasopharyngeal aspirate for pertussis

H pH studies

I Urine metabolic screen

66CSF culture

66Blood cultures

66Urine metabolic screen

 

Stats

 

Comments

 

Web links

Question statistics

Average score

64.40%

Times answered

294

Average user rating

4

 

My account Sign out

Page 79: cadiology 1.pdf

© 2010 BMJ Publishing Group Ltd . All rights reserved . Terms and

conditions . Privacy policy . A A A

Correct

The 5 month old presents with apnoea associated with mild

infection and hypoglycaemia. This points to a metabolic cause,

and a metabolic screen should be most helpful.

Apnoea can be caused by 3 main groups of conditions:

infections, obstructions, or toxins/drugs.

Rate this question

Next question